QUAN ĐẾN PHÂN HOẠCH TẬP HỢP

Một phần của tài liệu (LUẬN văn THẠC sĩ) một số dạng toán số học tổ hợp (Trang 39 - 60)

HỢP

Lý thuyết mở đầu về phân hoạch tập hợp tỏ ra khá đơn giản, nhưng những áp dụng của nó rất phong phú. Nhiều bài toán khó trong các đề thi chọn học sinh giỏi các cấp và Olympic Toán quốc tế thường được giải quyết khá nhanh gọn và độc đáo nhờ váo việc áp dụng phương pháp phân hoạch tập hợp.

Khái niệm phân hoạch tập hợp

Các tập hợp khác rỗng A1, A2, ..., Ak được gọi là một phân hoạch của tập hợp A nếu      A = A1 ∪A2 ∪...∪Ak, Ai ∩Aj = ∅;∀i, j ∈ {1,2, ..., k}, i 6= j.

Mỗi tập con Ai, với i = 1, k, được gọi là một thành phần của phân hoạch.

liệu [3].

3.1 Dạng toán yêu cầu nêu phân hoạch tập hợp

Bài toán 3.1.1. Giả sử c là một số hữu tỉ dương khác 1. Chứng minh rằng có thể phân hoạch tập hợp các số nguyên dương thành hai tập khác nhau A, B sao cho x

y 6= c, với mọi x, y cùng thuộc tập A hoặc cùng thuộc tập B.

Lời giải. Đặt c = p

q, với p, q là hai số nguyên tố cùng nhau.

Ta thấy số không chia hết cho p và không chia hết cho q thì đặt thêm chúng vào tập A hoặc tập B không ảnh hưởng gì đến tính chất của hai tập hợp này và thỏa mãn điều kiện của bài toán đặt ra.

Xét các số chia hết chophoặc chia hết choq. Chúng có dạngkpmqn, trong đó k không chia hết cho p và q.

Ta chia các số đó thành các nhóm dựa theo bậc (m+n) của chúng. Vấn đề còn lại là cần tiếp tục phân các số có cùng bậc vào các tập hợp khác nhau, bởi vì chính những số này nếu ở chung một tập hợp thì có thể xảy ra trường hợp x

y = c.

Để thỏa mãn điều kiện của bài toán, ta cần tránh thương x

y có kết quả mà trong đó p có bậc 1, vì lúc đó có thể x y = p q = c. Do đó, ta chỉ cần phân các số có cùng bậc p chẵn vào một tập hợp còn bậc p lẻ vào một tập hợp khác.

Ví dụ với bậc 5, nếu kp0q5, kp2q3, kp4q thuộc tập A

p, q.

Theo cách chia như trên ta phân hoạch được toàn bộ tập hợp các số nguyên dương thành hai tập hợp thỏa mãn điều kiện đề bài.

Bài toán 3.1.2. Tìm số các phân hoạch tập hợp A= {1,2,3, ..., n} thành ba tập hợp con A1, A2, A3(các tập này có thể là tập rỗng) sao cho các điều kiện sau được thỏa mãn:

(i) Sau khi sắp xếp các phần tử của A1, A2, A3 theo thứ tự tăng dần thì hai phần tử liên tiếp luôn có tính chẵn lẻ khác nhau;

(ii) Nếu cả ba tập A1, A2, A3 đều không rỗng thì có đúng một tập có số nhỏ nhất là số chẵn.

Lời giải. Ta có thể giả thiết thêm: (iii) 1 ∈ A1 và số nhỏ nhất trong A2 bé hơn số nhỏ nhất trong A3.

Ta xây dựng các phân hoạch bằng cách xếp lần lượt các số1,2, ..., n

vào A1, A2, A3 theo các điều kiện (i),(ii),(iii).

Ta có 1 ∈ A1, còn số 2 có hai khả năng: 2 ∈ A1 hoặc 2 ∈ A2 (do điều kiện (iii)).

Nếu A2 và A3 còn là tập rỗng thì các số tiếp theo, do điều kiện

(iii), đều chỉ có hai cách sắp xếp: vào A1 hoặc A2.

Sau khi phần tử đầu tiên của A2 được xếp, thì số tiếp theo chỉ có hai cách xếp: vào A2 hoặc vào A3 (vì số đó có cùng tính chẵn, lẻ với phần tử cuối của A1 lúc đó).

Khi A3 còn là tập rỗng thì, do điều kiện (i), (ii), các số tiếp theo cũng chỉ có hai cách xếp vào hai trong ba tập A1, A2, A3.

tiếp bằng phép quy nạp rằng, mỗi số luôn có thể và chỉ có thể xếp vào hai trong ba tập A1, A2, A3.

Thật vậy, giả sử đến một bước nào đó, số k có thể xếp vào một trong hai tập Ai1, Ai2 và không thể xếp vào Ai3, với (i1, i2, i3) là một hoán vị nào đó của (1,2,3), nghĩa là k khác tính chẵn, lẻ với hai số cuối Ai1,

Ai2 và cùng tính chẵn, lẻ với số cuối của Ai3 lúc đó.

Không mất tính tổng quát, ta giả sử xếp k vào Ai1 thì ở bước tiếp theo số k + 1 chỉ có thể xếp vào Ai1 hoặc Ai3 (do điều kiện (i)), nghĩa là có đúng hai khả năng sắp xếp đối với số (k+ 1).

Tóm lại, với mỗi số trong tập {1,2, ..., n}, trừ số 1, đều cho ta hai khả ngăng sắp xếp.

Vậy có 2n−1 phân hoạch tập {1,2, ..., n}vào ba tập A1, A2, A3 theo yêu cầu bài toán.

Bài toán 3.1.3. Cho tập hợp M gồm n số dương a1, a2, ..., an. Xét tất cả các tập con Ti khác rỗng của M. Gọi si là tổng các số thuộc một tập hợp con Ti nói trên. Chứng minh rằng có thể chia tập hợp tất cả các số si

được thành lập như vậy thành n tập hợp con khác rỗng không giao nhau sao cho tỉ số của hai số bất kì thuộc cùng một tập hợp con vừa được phân chia không vượt quá 2.

Lời giải. Không mất tính tổng quát, giả sử a1 6 a2 6 ... 6 an. Đặt S0 = 0, Sm = a1 +a2 +...+am, với 1 6 m 6 n.

Gọi P = {si\Ti ⊆ M}.

Kí hiệu Pm = {s ∈ P\Sm−1 < s 6 Sm}, với m ∈ {1,2, ..., n}. Ta chứng minh rằng cách chia P thành các tập Pm như trên thỏa mãn điều

kiện bài toán.

Muốn vậy ta chỉ cần chứng minh: Nếu b ∈ Pm thì 2b > Sm. Thật vậy, do b > Sm−1 = a1 + a2 + ...+am−1 và b =

h

X

k=1

aik, nên tồn tại ik, với k = 1,2, ..., h sao cho ik > m.

Vậy b> aik > am = Sm−Sm−1 > Sm −b hay 2b > Sm. Ta có điều phải chứng minh.

3.2 Dạng toán giải bằng phương pháp phân hoạch tập hợp

Bài toán 3.2.1. Cho A là tập hợp gồm 16 số nguyên dương đầu tiên. Hãy tìm số nguyên dương k nhỏ nhất có tính chất: Trong mỗi tập con có k phần tử của tập hợp A đều tồn tại hai số phân biệt a và b sao cho a2 + b2 là một số nguyên tố.

Lời giải. Giả sử k là số nguyên dương sao cho trong mỗi tập con có k phần tử của tập A đều tồn tại hai số phân biệt a và b sao cho a2+b2

là một số nguyên tố.

Xét tập con T gồm tất cả các số chẵn thuộc tập A. Dễ thấy T có

8 phần tử và với a, b tùy ý thuộc T luôn có a2 +b2 là một hợp số. từ đó suy ra k > 9.

Bằng cách tính và kiểm tra trực tiếp tất cả các tổng a2 + b2, với

a, b ∈ A, ta thu được một phân hoạch gồm 8tập hợp con của tập Amà mỗi tập con gồm hai phần tử là hai số có tổng bình phương là một số nguyên tố

A = {1,4}∪{2,3}∪{5,8}∪{6,11}∪{7,10}∪{9,16}∪{12,13}∪{14,15}. Theo nguyên lí Dirichlet, trong 9 phần tử tùy ý của tập A phải có

hai phần tử thuộc cùng một tập con trong phân hoạch nói trên. Nói cách khác, trong mỗi tập con có 9 phần tử của tập A đều tồn tại hai số phân biệt a, b mà a2 +b2 là một số nguyên tố.

Từ các kết quả trên, ta suy ra min (k) = 9.

Bài toán 3.2.2. Cho p, q nguyên lẻ nguyên tố cùng nhau. Chứng minh

p−1 2 X i=1 iq p + q −1 2 X j=1 jp q = p−1 2 q −1 2 .

Lời giải. Giả sử A = {(i;j)\1 6 i 6 p−1

2 ,16 j 6 q−1 2 }. Khi đó |A| = p−1 2 q −1 2 . Ta phân tập A thành các tập hợp: A1 = {(i;j)\qi > pj}, A2 = {(i;j)\qi < pj}, A3 = {(i;j)\qi = pj}. Ta có |A3| = 0. Mặt khác |A1| = p−1 2 X i=1 iq p 0 |A2| = q−1 2 X j=1 jp q .

Vì |A| = |A1|+|A2| nên ta có điều phải chứng minh.

Bài toán 3.2.3. Đối với mỗi số tự nhiên n ∈ N, hãy tìm số tự nhiên

k ∈ N lớn nhất thỏa mãn điều kiện: trong tập gồm n phần tử có thể chọn ra được k tập con khác nhau, mà hai tập bất kì trong các tập con này đều có giao khác rỗng.

1) Chứng minh k > 2n−1.

Ta cố định phần tử a1 và chỉ xét các tập con A1, A2, ..., Ak có chứa

a1. Khi đó, số tập con này bằng đúng số tập con của tập {a2, a3, ..., an}. Mà ∀i, j ∈ {1,2, ..., k} đều có a1 ∈ Ai∩Aj. Nên k > 2n−1.

2) Chứng minh k 6 2n−1.

Giả sử chọn đượck > 2n−1 tập con của tập X, mà giao của hai tập bất kì trong các tập được chọn ra đều khác rỗng.

Chia tất cả 2n tập con của tập X thành 2n−1 cặp sao cho trong mỗi cặp đều gồm tập con và phần bù của nó.

Vì số tập con chọn ra lớn hơn 2n−1 nên theo nguyên lí Dirichlet, phải có ít nhất hai tập con đã chọn lập thành một cặp.

Do đó giao của hai tập này bằng rỗng, mâu thuẫn với tính chất của các tập hợp đã chọn ra.

Vậy k 6 2n−1.

Tóm lại, số tự nhiên k lớn nhất thõa mãn điều kiên bài toán là

k = 2n−1.

Bài toán 3.2.4. Khai triển f (x) = 1 +x+x2 +...+ x1010, ta thu được đa thức f (x) = a0 +a1x+a2x2 +...+a100x100.

Tính tổng S = a0 +a1 +...+a10.

Lời giải. Trước hết ta chứng minh bổ đề sau. Bổ đề. Cho hai số tự nhiên n, k. Xét tập hợp

Hn,k = {(x0, x1, ..., xn)\x0, x1, ..., xn ∈ N, x0 + x1 + ...+xn = k}.

Chứng minh. Ta chứng minh bổ đề trên bằng phương pháp quy nạp theo n.

Khi n = 0 thì H0,k = {k}, |H0,k| = 1 = Ck0: khẳng định đúng. Giả sử khẳng định đúng đến (n−1), với n > 1.

Xét phân hoạch Hn,k = B0 ∪B1 ∪...∪Bk, trong đó (x0, x1, ..., xn) ∈ Bj

nếu xn = j, với j ∈ {0,1,2, ..., k}.

Theo giả thiết quy nạp ta có: |Bj| = |Hn−1,k−j| = Cnn−−1+k1 −j,∀j = 0,1,2, ..., k. Dùng công thức Cmi−1 + Cmi = Cm+1i , ta có |Hn,k| = k X j=0 |Bj| = k X j=0 Cnn−−1+k1 −j = k X j=0 Cn+kn −j −Cnn−1+k−j = Cn+kn . Do đó ta có điều phải chứng minh.

Lưu ý: |Hn,k| = Cn+kn −Cnn−1 = Cn+kn , với quy ước rằng Cnn−1 = 0. Trở lại bài toán ở dạng tổng quát sau:

Vớim, n ∈ N, khai triểnf(x) = 1 +x+x2 +...+xmn+1 ta được đa thức f(x) =a0 + a1x+a2x2 +...+am(n+1)xm(n+1).

Viết cụ thể khai triển của f(x), ta có

ai = |Hn,i| = Cn+in , ∀i = 0,1,2, ..., m, |Hn,i| > ai, nếu i > m + 1. Nói riêng: a0+a1+a2+...+am = m X i=0 Cn+in = m X i=0 Cn+1+in+1 −Cn+in+1 = Cn+1+mn+1 . Bài toán đã cho là trường hợp n+ 1 = m = 10.

Do đó S = C2010.

và Ai, với i = 1, m, là các tập con khác nhau và gồm ít nhất hai phần tử của S sao cho từ các quan hệ Ai ∩Aj 6= ∅, Aj ∩Ak 6= ∅, Ak ∩Ai 6= ∅, ta suy ra được Ai∩ Aj ∩Ak 6= ∅. Chứng minh rằng: m 6 2n−1 −1.

Lời giải. Ta chứng minh bằng phương pháp quy nạp theo n. Hiển nhiên, phát biểu ở đề bài đúng khi n = 2.

Giả sử n > 2 và phát biểu đúng với mọi số nguyên k < n. Ta xét hai trường hợp:

Trường hợp 1: Không tồn tại i, j, với i = 1, m để cho Ai∪Aj = S

và |Ai ∩Aj| = 1.

Gọi x là phần tử tùy ý của S. Số tất cả các tập Ai không chứa x

lớn nhất bằng 2n−2 −1, theo giả thiết quy nạp. Số các tập con chứa x của S là 2n−1.

Nếux ∈ Ai thì sẽ không tồn tại sốj nào để choAj = (S \Ai)∪{x}, bằng không thì phải có |Ai ∩Aj| = 1.

Do vậy, quá lắm là một nửa các tập con chứa x của S xuất hiện dưới dạng các tập Ai.

Như thế, số lớn nhất các tập Ai là 2n−2 −1 + 2n−2 = 2n−1 −1. Trường hợp 2: Tồn tại một phần tử x ∈ S sao cho A1∪A2 = S và

A1 ∩A2 = {x}.

Đặt |A1| = r+ 1 và |A2| = s+ 1. Khi đó r +s = n−1.

Theo giả thiết quy nạp, số lớn nhất của các tậpAi sao cho Ai ⊆ A1

là 2r −1.

Nếu Ai không phải là tập con của A1 và A2 thì A1 ∩ Ai 6= ∅,

A2 ∩ Ai 6= ∅. Vì A1 ∩ A2 =6 ∅, nên A1 ∩ A2 ∩ Ai 6= ∅. Như vậy ta có

A1 ∩A2 ∩Ai = {x}.

Do đó, Ai = {x} ∪(Ai \A1)∪(Ai \A2).

Ngoài ra, do các tập khác rỗng (Ai \A1) và (Ai \A2) có thể được chọn tương ứng theo (2s−1) và (2r −1) cách, nên số lớn nhất các tập này là (2s −1) (2r −1).

Công thêm các kết kết quả riêng này vào, ta nhận được số lớn nhất các tập Ai là 2n−1 −1.

Bài toán 3.2.6. Chứng minh rằng nếu gọi f (r, n) là số phân hoạch của n thành dạng b0 +b1 + b2 + ...+bs với 0 ≤ i ≤ s−1 và bi > rbi+1, r là một số nguyên dương nào đó thì tồn tại số nguyên dương n mà f (r, n)

khác số phân hoạch của n thành các phần từ một tập hợp các số nguyên bất kỳ, trừ khi r = 1.

Lời giải. Ta chứng minh bài toán này bằng phản chứng.

Giả sử tồn tại sốr ≥ 2sao cho có một tập Sr các số nguyên dương. Nếu gọi số phân hoạch của n thành các phần lấy từ tập Sr là g(r, n) thì

f (r, n) = g(r, n) với mọi n.

Ta có f (r, 1) = 1 nên 1∈ Sr. Vì nếu không thì g(r, 1) = 0. Tiếp theo f (r, 2) = 1 nên 2∈/ Sr vì nếu không thì g(r, 2) = 2.

Do f (r, 3) = f (r, 4) = ... = f (r, r+ 1) = 1 nên ta có được

3, 4, 5, ..., r+ 1 ∈/ Sr.

Ta cóf (r, r+ 2) = 2do phân hoạch củar+2thỏa mãn bi > rbi+1

Suy rar+ 2 ∈ Sr vì nếu không thì g(r, r+ 2) = 1. Ta có bảng thể hiện các giá trị như sau: f (n), g(r, n) nếu n /∈ Sr, g(r, n) nếu n ∈ Sr

1 1 0 1 1 ∈ S −r 2 1 1 2 2 ∈/ S −r 3 1 1 2 3 ∈/ S −r . . . . r + 1 1 1 2 r + 1 ∈/ S −r r + 2 1 1 2 r + 2 ∈/ S −r r + 3 2 2 3 r + 3 ∈/ S −r . . . . 2r + 1 2 2 3 2r + 2 ∈/ S −r 2r + 2 3 2 3 2r + 3 ∈/ S −r 2r + 3 3 4 5 2r + 3 ∈/ S −r

Tiếp theo ta xác định giá trị nhỏ nhất của n để f (r, n) = 3. Điều này xảy ra khi n = b0 + b1 + 1 với b0 > rb1 và b1 > r hoặc

n= b0 + 2 với b0 > 2r.

Giá trị n trong trường hợp thứ nhất nhỏ nhất là

n = r2 +r + 1+ (r + 1) + 1 =r2 + 2r + 3,

trường hợp thứ hai là n = (2r + 1) + 2 = 2r + 3.

Do 2r + 3 < r2 + 2r + 3, ∀r > 0 nên n = 2r + 3 là giá trị nhỏ nhất của n mà f (r, n) = 3. Trong trường hợp này, các phân hoạch của n

Một phần của tài liệu (LUẬN văn THẠC sĩ) một số dạng toán số học tổ hợp (Trang 39 - 60)

Tải bản đầy đủ (PDF)

(60 trang)